George has a total of B books in his library

This topic has expert replies
Senior | Next Rank: 100 Posts
Posts: 83
Joined: Mon Jul 24, 2017 8:16 am
Followed by:1 members

Timer

00:00

Your Answer

A

B

C

D

E

Global Stats

George has a total of B books in his library, 25 of which are hardcover fiction books. What is the value of B?

(1) 40 of the B books are fiction and the rest are nonfiction
(2) 60 of the B books are hardcovers and the rest are paperbacks

Official Guide question
Answer: E

GMAT/MBA Expert

User avatar
GMAT Instructor
Posts: 3008
Joined: Mon Aug 22, 2016 6:19 am
Location: Grand Central / New York
Thanked: 470 times
Followed by:34 members

by Jay@ManhattanReview » Thu Jul 27, 2017 11:02 pm

Timer

00:00

Your Answer

A

B

C

D

E

Global Stats

jjjinapinch wrote:George has a total of B books in his library, 25 of which are hardcover fiction books. What is the value of B?

(1) 40 of the B books are fiction and the rest are nonfiction
(2) 60 of the B books are hardcovers and the rest are paperbacks

Official Guide question
Answer: E
We have the total number of books = B;

Say,
Number of Hardcover books = H;
Number of Hardcover Fiction books = HF;
Number of Hardcover Non-fiction books = HNF;

Number of Paperback books = P;
Number of Paperback Fiction books = PF;
Number of Paperback Non-fiction books = PNF

Thus,
B = H + P ---(1)

B = [HF + HNF] + [PF + PNF] ---(2)

B = [25 + HNF] + [PF + PNF] ---(2)

We have to get the value of B.

Statement 1: 40 of the B books are fiction and the rest are nonfiction.

=> HF + PF = 40 and HNF + PNF = B - 40
=> PF = 40 - 25 = 15

From eqn (2), we have

B = [25 + HNF] + [15 + PNF] ---(2)
B = 40 + HNF + PNF
B = 40 + B - 40
B = B. Can't get B. Insufficient.

Statement 2: 60 of the B books are hardcovers and the rest are paperbacks.

=> H = 60 => H = HF + HNF => 60 = 25 + HNF => HNF = 35

From eqn (1), we have

B = H + P ---(1)

B = 40 + P. We do not know the value of P. Insufficient.

Statement 1 and 2:

Even after combining both the statement, we cannot get the value of PNF, thus the value of B. Insufficient.

The correct answer: E

Hope this helps!

Download free ebook: Manhattan Review GMAT Quantitative Question Bank Guide

-Jay
_________________
Manhattan Review GMAT Prep

Locations: New York | Singapore | Doha | Lausanne | and many more...

Schedule your free consultation with an experienced GMAT Prep Advisor! Click here.

GMAT/MBA Expert

User avatar
Elite Legendary Member
Posts: 10392
Joined: Sun Jun 23, 2013 6:38 pm
Location: Palo Alto, CA
Thanked: 2867 times
Followed by:511 members
GMAT Score:800

by [email protected] » Mon Mar 26, 2018 3:49 pm

Timer

00:00

Your Answer

A

B

C

D

E

Global Stats

Hi All,

We're told that George has a total of B books in his library, 25 of which are HARDCOVER FICTION books. We're asked for the value of B. Based on the information given so far, it's likely that there will be hardcover and 'not' hardcover (meaning 'paperback') books as well as fiction and 'not' fiction (meaning 'nonfiction) books. This implies that there are many variables to keep track of - and we'll need a LOT of specific information for either Fact to be considered Sufficient.

1) 40 of the B books are fiction and the rest are nonfiction

With Fact 1, we now know that 40 of the books are FICTION (which includes the 25 that are HARDCOVER FICTION). Thus, there are 40 - 25 = 15 paperback fiction books, but we still don't know how many non-fiction books there are (of either type).
Fact 1 is INSUFFICIENT

2) 60 of the B books are hardcovers and the rest are paperbacks

With Fact 2, we now know that 60 of the books are HARDCOVER (which includes the 25 that are HARDCOVER FICTION). Thus, there are 60 - 25 = 35 hardcover nonfiction books, but we still don't know how many paperback books there are (of either type).
Fact 2 is INSUFFICIENT

Combined, we know there are...
25 Hardcover Fiction books
15 Paperback Fiction books
35 Hardcover Non-Fiction books
X Paperback Non-Fiction books

Since we don't know the number of Paperback Non-Fiction books, there's not enough information to answer the question.
Combined, INSUFFICIENT.

Final Answer: E

GMAT assassins aren't born, they're made,
Rich
Contact Rich at [email protected]
Image

GMAT/MBA Expert

User avatar
GMAT Instructor
Posts: 7223
Joined: Sat Apr 25, 2015 10:56 am
Location: Los Angeles, CA
Thanked: 43 times
Followed by:29 members

Timer

00:00

Your Answer

A

B

C

D

E

Global Stats

jjjinapinch wrote:
Thu Jul 27, 2017 12:11 pm
George has a total of B books in his library, 25 of which are hardcover fiction books. What is the value of B?

(1) 40 of the B books are fiction and the rest are nonfiction
(2) 60 of the B books are hardcovers and the rest are paperbacks

Official Guide question
Answer: E
Solution:

Question Stem Analysis:

We need to determine the value of B, the number of books in George’s library. (Note: Here we are assuming all the books in George are either fiction or nonfiction, and either hardcover or paperback.)

Statement One Alone:

Since 40 books are fiction books and 25 are hardcover fiction books, 40 - 25 = 15 books are paperback fiction books. However, we can’t determine the value of B, the number of books in George’s library. Statement one alone is not sufficient.

Statement Two Alone:

Since 60 books are hardcovers and 25 are hardcover fiction books, 60 - 25 = 35 books are hardcover nonfiction books. However, we can’t determine the value of B, the number of books in George’s library. Statement two alone is not sufficient.

Statements One and Two Together:

From the two statements, we can see that there are 25 hardcover fiction books, 15 paperback fiction books, and 35 hardcover nonfiction books. However, since we still don’t know the number of paperback nonfiction books, we can’t determine the value of B, the number of books in George’s library. Both statements together are still not sufficient.

Answer: E

Scott Woodbury-Stewart
Founder and CEO
[email protected]

Image

See why Target Test Prep is rated 5 out of 5 stars on BEAT the GMAT. Read our reviews

ImageImage